Integral maraton !

Her kan brukere av forum utfordre hverandre med morsomme oppgaver og nøtter man ønsker å dele med andre. Dette er altså ikke et sted for desperate skrik om hjelp, de kan man poste i de andre forumene, men et sted for problemløsing på tvers av trinn og fag.

Moderatorer: Vektormannen, espen180, Aleks855, Solar Plexsus, Gustav, Nebuchadnezzar, Janhaa

Markus
Fermat
Fermat
Innlegg: 767
Registrert: 20/09-2016 13:48
Sted: NTNU

Mattegjest skrev:Oppfølgar:

Rekn ut det bestemte integralet (e[tex]^{-x}[/tex] * x[tex]^{-1}[/tex] * (1 - cos( x ) ) dx frå 0 til + uendeleg .
Definer $$I(a)=\int_0^\infty \frac{1-\cos(x)e^{-ax}}{ x} \, \text{d}x$$ Da er $$I'(a)=\int_0^\infty e^{-ax}(\cos(x)-1) \, \text{d}x$$ Vi ser på dette integralet uten å tenke på grensene. Ved delvis intgrasjon med $u=\cos(x)-1$ og $v'=e^{-ax}$, fås $$\int e^{-ax}(\cos(x)-1) \, \text{d}x = -\frac{e^{-ax}(\cos(x)-1)}{a} - \frac1a \int e^{-ax}\sin(x)$$ Delvis integrasjon på det siste leddet med $u=\sin(x)$ og $v'=e^{-ax}$ gir $$\int e^{-ax}\sin(x) \, \text{d}x = -\frac{e^{-ax}\sin(x)}{a}+\int \frac{e^{-ax}\cos(x)}{a} \, \text{d}x$$ Delvis integrasjon en gang til på det siste leddet i det siste integralet med $u=\cos(x)$ og $v'=\frac{e^{-ax}}{a}$ gir at $$\int e^{-ax}\sin(x) \, \text{d}x = -\frac{e^{-ax}\sin(x)}{a} - \frac{e^{-ax}\cos(x)}{a^2} - \frac{1}{a^2}\int e^{-ax}\sin(x) \, \text{d}x \\ \therefore \left(1+\frac{1}{a^2} \right) \int e^{-ax}\sin(x) \,\text{d}x = \frac{-ae^{-ax}\sin(x) - e^{-ax}\cos(x)}{a^2} \\ \therefore \int e^{-ax}\sin(x) \, \text{d}x = \frac{ae^{-ax}\sin(x) - e^{-ax}\cos(x)}{a^2+1}$$ Som gir at $$I'(a)=\left[\frac{e^{-ax}\left(a\sin(x)-a^2\cos(x) + a^2 +1 \right)}{a^3+a} \right ]_0^\infty = - \frac{1}{a^3+a}$$ Nå er $$I(a)=\int - \frac{1}{a^3+a} \, \text{d}a = \int - \frac{1}{\left(1+\frac{1}{a^2} \right )a^3}$$ La $u=\frac{1}{a^2} + 1$, da er $\text{d}a = -\frac{a^3}{2} \, \text{d}u$ så $$I(a)=\frac12 \int \frac1u \, \text{d}u = \frac12 \ln |u| + C = \frac12 \ln|\frac{1}{a^2}+1|+C$$ For å finne integrasjonskonstanten $C$ lar vi $a\to \infty$ i det originale uttrykket for $I(a)$ og ser at vi da får $\lim_{a\to \infty} I(a) = 0$. Gjør vi det samme i det siste uttrykket får vi også at $I(a) \to 0$. Altså må integrasjonskonstanten være $0$ og vi får endelig at $$\int_0^\infty \frac{(1-\cos(x))e^{-x}}{x} = I(1)=\frac12 \ln(2)$$

Oppfølger: $$\int \cos^4(x) - \sin^4(x) \, \text{d}x$$
Sist redigert av Markus den 17/06-2018 21:28, redigert 1 gang totalt.
Mattebruker

Hadde tenkt å legge inn eit passande hint ( derivasjon under integralteiknet ) , men Markus løyste problemet på
sjølvstendig grunnlag.
Kay
Abel
Abel
Innlegg: 684
Registrert: 13/06-2016 19:23
Sted: Gløshaugen

Markus skrev:
Oppfølger: $$\int \cos^4(x) - \sin^4(x) \, \text{d}x$$
[tex]\int \cos^4(x)-\sin^4(x)dx=\int \cos(2x)dx=\frac{1}{2}\sin(2x)[/tex]


Oppfølger [tex]\int_{-\infty}^{+\infty} e^{-x^2}cos(x)dx[/tex]
Markus
Fermat
Fermat
Innlegg: 767
Registrert: 20/09-2016 13:48
Sted: NTNU

Kay skrev:Oppfølger [tex]\int_{-\infty}^{+\infty} e^{-x^2}cos(x)dx[/tex]
La $$I(a)=\int_{-\infty}^{\infty} e^{-x^2} \cdot \cos(ax) \, \text{d}x$$ Da er $$I'(a)= \int_{-\infty}^{\infty} -xe^{-x^2} \cdot \sin(ax) \, \text{d}x$$ Utfører vi nå delvis integrasjon på $I'(a)$ med $u'=-xe^{-x^2}$ og $v=\sin(ax)$ fås $$I'(a)= \left[\frac{1}{2}e^{-x^2} \cdot \sin(ax) \right]_{-\infty}^\infty - \int_{-\infty}^{\infty} \frac{1}{2}e^{-x^2} \cdot (a \cdot \cos(ax)) \, \text{d}x = -\frac{a}{2}\int_{-\infty}^\infty e^{-x^2}\cdot \cos(ax) \, \text{d}x$$ Legg merke til at det siste uttrykket er $I(a)$, altså har vi at $I'(a)=-\frac{a}{2}I(a)$, som er en seperabel difflikning i forkledning; $\frac{y'}{y}=-\frac{a}{2}$. Vi får $$\int \frac{1}{y} \frac{\text{d}y}{\text{d}a} \, \text{d}a = \int -\frac{a}{2} \, \text{d}a \, \therefore \, \ln(y) = -\frac{a^2}{4} + C \, \therefore \, I(a)=y=Ce^{-\frac{a^2}{4}}$$ Observer nå at $$I(0)=C=\int_{-\infty}^\infty e^{-x^2} = \sqrt{\pi}$$ som er et kjent resultat (kontekst: Gaussian integral), så utelater det da flere sikkert er kjent med det fra før. Fra den originale definisjonen på $I(a)$ er vårt ønskede integral $I(1)$, som gir endelig at $$\int_{-\infty}^{\infty} e^{-x^2}\cos(x) = I(1) = \sqrt{\pi}e^{-\frac14}$$

Oppfølger: $$\int_{-1}^{1} \frac{\sin(x)}{x^4+x^2+1} \, \text{d}x$$
Sist redigert av Markus den 18/06-2018 19:51, redigert 1 gang totalt.
MatIsa
Dirichlet
Dirichlet
Innlegg: 150
Registrert: 12/06-2013 12:09
Sted: Trondheim

Kay skrev:Oppfølger [tex]\int_{-\infty}^{+\infty} e^{-x^2}cos(x)dx[/tex]
Litt usikker på om dette er gyldig, men mener å ha sett noe lignende før:
$\int_{-\infty}^\infty e^{-x^2}\cos(x)~{\rm d}x = \mathfrak{R}\left[\int_{-\infty}^\infty e^{-x^2+ix}~{\rm d}x\right] = \mathfrak{R}\left[\int_{-\infty}^\infty e^{-(x-i/2)^2-1/4}~{\rm d}x\right] = \mathfrak{R}\left[e^{-1/4}\int_{-\infty}^\infty e^{-(x-i/2)^2}~{\rm d}x\right] = e^{-1/4}\sqrt{\pi}$

Edit: Var visst litt sent ute der :oops:
Mattebruker

Integranden sin( x ) /(x[tex]^4[/tex] + x[tex]^2[/tex] + 1) er åpenbart ein odde funksjon( teljar sin( x ) er odde ,

medan nemnar er jamn ) . Da blir det bestemte integralet lik 0 ettersom nedre og øvre integrasjonsgrense ligg like langt
frå origo( a = - b).


Oppfølgar:

Rekn ut trippelintegralet (x[tex]^2[/tex] + y[tex]^2[/tex] + z[tex]^2[/tex] + a[tex]^2[/tex])[tex]^{-2}[/tex]dxdydz , a > 0

over D ={(x,y,z) : x > 0 , y > 0 og z > 0 }
Kay
Abel
Abel
Innlegg: 684
Registrert: 13/06-2016 19:23
Sted: Gløshaugen

MatIsa skrev:
Kay skrev:Oppfølger [tex]\int_{-\infty}^{+\infty} e^{-x^2}cos(x)dx[/tex]
Litt usikker på om dette er gyldig, men mener å ha sett noe lignende før:
$\int_{-\infty}^\infty e^{-x^2}\cos(x)~{\rm d}x = \mathfrak{R}\left[\int_{-\infty}^\infty e^{-x^2+ix}~{\rm d}x\right] = \mathfrak{R}\left[\int_{-\infty}^\infty e^{-(x-i/2)^2-1/4}~{\rm d}x\right] = \mathfrak{R}\left[e^{-1/4}\int_{-\infty}^\infty e^{-(x-i/2)^2}~{\rm d}x\right] = e^{-1/4}\sqrt{\pi}$

Edit: Var visst litt sent ute der :oops:
For det det er verdt så var begge iallefall riktig. Jeg må også si meg imponert med begge løsningene, en differensial og en ved hjelp av kompleks er flott (for øvrig imponert over at du tok alt på en eneste linje).
DennisChristensen
Grothendieck
Grothendieck
Innlegg: 826
Registrert: 09/02-2015 23:28
Sted: Oslo

Mattegjest skrev: Oppfølgar:

Rekn ut trippelintegralet (x[tex]^2[/tex] + y[tex]^2[/tex] + z[tex]^2[/tex] + a[tex]^2[/tex])[tex]^{-2}[/tex]dxdydz , a > 0

over D ={(x,y,z) : x > 0 , y > 0 og z > 0 }
$$I = \iiint_D \left(x^2 + y^2 + z^2 + a^2\right)^{-2}\, \text{d}x\,\text{d}y\,\text{d}z,$$
der $a>0$ og $D = \{ (x,y,z)\in\mathbb{R}^3 | x,y,z > 0\}.$

Vi introduserer sfæriske polarkoordinater: $$x = r\sin\varphi\cos\theta,$$ $$y=r\sin\varphi\sin\theta,$$ $$z = r\cos\varphi,$$ slik at integralet vårt blir til
$$\int_{\varphi = 0}^{\frac{\pi}{2}}\int_{\theta = 0}^{\frac{\pi}{2}}\int_{r=0}^{\infty}\frac{r^2\sin\varphi}{\left(r^2 + a^2\right)^2}\, \text{d}r\,\text{d}\theta\,\text{d}\varphi = \frac{\pi}{2}\left[-\cos\varphi\right]_{\varphi = 0}^{\frac{\pi}{2}}\int_{r=0}^{\infty}\frac{r^2}{\left(r^2 + a^2\right)^2}\,\text{d}r = \frac{\pi}{2}\int_{r=0}^{\infty}\frac{r^2}{\left(r^2+a^2\right)^2}\,\text{d}r.$$

Vi evaluerer dette integralet via Cauchy's Residue Theorem. Integrér $f(z) = \frac{z^2}{(z^2+a^2)^2}$ mot klokken rundt $\Gamma_R$, der $\Gamma_R$ er standardkonturen med radius $R$. Integralet langs halvsirkelen går mot $0$ når $R\rightarrow\infty$ ettersom integranden er $O(\frac{1}{R^2})$. Den eneste polen inni $\Gamma_R$ er $z=ia$ (ettersom $a>0$), av orden $2$, og vi ser at $$\text{Res}(f;ia) = \lim_{z\rightarrow ia}\frac{\text{d}}{\text{d}z}\left[(z-ia)^2f(z)\right] = \lim_{z\rightarrow ia} \frac{\text{d}}{\text{d}z}\frac{z^2}{(z+ia)^2} = \lim_{z\rightarrow ia}\frac{2z(z+ia)^2 - 2z^2(z+ia)}{(z+ia)^4} = \frac{2ia(2ia)^2 - 2(ia)^2(2ia)}{(2ia)^4} = \frac{-8ia^3 + 4ia^3}{16a^4} = -\frac{1}{4a}i,$$ så Cauchy gir at $$\int_{-\infty}^{\infty}\frac{x^2}{(x^2+a^2)^2}\,\text{d}x = \lim_{R\rightarrow \infty}\int_{\Gamma_R}f(z)\, \text{d}z = 2\pi i\left(-\frac{1}{4a}i\right) = \frac{\pi}{2a}.$$

Dermed får vi til slutt at $$I = \frac{\pi}{2}\int_{r=0}^{\infty}\frac{r^2}{\left(r^2+a^2\right)^2}\,\text{d}r = \frac{\pi}{4}\int_{-\infty}^{\infty}\frac{x^2}{\left(x^2+a^2\right)^2}\, \text{d}x = \frac{\pi}{4}\times\frac{\pi}{2a} = \frac{\pi^2}{8a}. $$

Oppfølger: La $a,b,c > 0$ være konstanter. Evaluér integralet $$\iiint_D \left(ax +by + cz\right)^4 \, \text{d}V,$$ der $D = \{(x,y,z)\in\mathbb{R}^3 | x^2 + y^2 + z^2 \leq 1\}$.
MatIsa
Dirichlet
Dirichlet
Innlegg: 150
Registrert: 12/06-2013 12:09
Sted: Trondheim

Kay skrev:For det det er verdt så var begge iallefall riktig. Jeg må også si meg imponert med begge løsningene, en differensial og en ved hjelp av kompleks er flott (for øvrig imponert over at du tok alt på en eneste linje).
Sier aldri nei til en liten tur innom kompleks analyse 8-)
DennisChristensen skrev:Oppfølger: La $a,b,c > 0$ være konstanter. Evaluér integralet $$\iiint_D \left(ax +by + cz\right)^4 \, \text{d}V,$$ der $D = \{(x,y,z)\in\mathbb{R}^3 | x^2 + y^2 + z^2 \leq 1\}$.
Etter å brukt divergensteoremet til å skrive om volumintegralet til et overflateintegral med $F = \dfrac{(ax+by+cz)^5}{15}\left(\dfrac1a, \dfrac1b, \dfrac1c\right)$, innså jeg at det fantes en langt enklere løsning: Integranden $(ax+by+cz)^4$ er konstant langs plan, og man kan evaluere integralet ved å integrere over alle plan som har avstand mindre enn 1 til origo. La $\alpha: ax+by+cz+w = 0$ være et plan. Da er avstanden (med fortegn) fra $\alpha$ til origo gitt ved $s(w) = w/\sqrt{a^2+b^2+c^2}$. Vi ønsker å se på planene som oppfyller $|s(w)| \leq 1$, altså $-\sqrt{a^2+b^2+c^2}\leq w \leq \sqrt{a^2+b^2+c^2}\Longrightarrow -w_0\leq w\leq w_0$, der $w_0 = \sqrt{a^2+b^2+c^2}$. Skriv $$Q = \iiint_D w^4~{\rm d}V = \int_{-w_0}^{w_0}~{\rm d}Q$$
der ${\rm d}Q$ er bidraget til $Q$ fra planet $\alpha$. Skjæringskurven mellom $\alpha$ og enhetskula er en disk med radius $r(w)$ og areal $A(w) = \pi r(w)^2$. Bidraget er da gitt ved ${\rm d}Q = w^4~{\rm d}V = w^4A(w)~{\rm d}s = \pi w^4r(w)^2~{\rm d}s$. Her er ${\rm d}s = \dfrac{{\rm d}s}{{\rm d}w}{\rm d}w = \dfrac{1}{w_0}{\rm d}w$. Fra figuren er det lett å se at $r(w)^2+s(w)^2=1\Longrightarrow r(w) = \sqrt{1-w^2/w_0^2}$. Dette gir da $$Q = \pi\int_{-w_0}^{w_0} w^4(1-w^2/w_0^2)({\rm d}w/w_0) = \dfrac{2\pi}{w_0}\int_0^{w_0}\left(w^4-w^6/w_0^2\right){\rm d}w = \dfrac{2\pi}{w_0}\left.\left(\dfrac{w^5}{5}-\dfrac{w^7}{7w_0^2}\right)\right|_0^{w_0}$$ $$~~~ = 2\pi\left(\dfrac{w_0^4}{5}-\dfrac{w_0^4}{7}\right) = \dfrac{4\pi}{35}w_0^4 = \dfrac{4\pi}{35}(a^2+b^2+c^2)^2$$

Oppfølger: $\int \dfrac{{\rm d}x}{x(x+1)(x+2)\cdots (x+n)}$
Figur.png
Figur.png (15.08 kiB) Vist 6747 ganger
DennisChristensen
Grothendieck
Grothendieck
Innlegg: 826
Registrert: 09/02-2015 23:28
Sted: Oslo

MatIsa skrev:Etter å brukt divergensteoremet til å skrive om volumintegralet til et overflateintegral med $F = \dfrac{(ax+by+cz)^5}{15}\left(\dfrac1a, \dfrac1b, \dfrac1c\right)$, innså jeg at det fantes en langt enklere løsning: Integranden $(ax+by+cz)^4$ er konstant langs plan, og man kan evaluere integralet ved å integrere over alle plan som har avstand mindre enn 1 til origo. La $\alpha: ax+by+cz+w = 0$ være et plan. Da er avstanden (med fortegn) fra $\alpha$ til origo gitt ved $s(w) = w/\sqrt{a^2+b^2+c^2}$. Vi ønsker å se på planene som oppfyller $|s(w)| \leq 1$, altså $-\sqrt{a^2+b^2+c^2}\leq w \leq \sqrt{a^2+b^2+c^2}\Longrightarrow -w_0\leq w\leq w_0$, der $w_0 = \sqrt{a^2+b^2+c^2}$. Skriv $$Q = \iiint_D w^4~{\rm d}V = \int_{-w_0}^{w_0}~{\rm d}Q$$
der ${\rm d}Q$ er bidraget til $Q$ fra planet $\alpha$. Skjæringskurven mellom $\alpha$ og enhetskula er en disk med radius $r(w)$ og areal $A(w) = \pi r(w)^2$. Bidraget er da gitt ved ${\rm d}Q = w^4~{\rm d}V = w^4A(w)~{\rm d}s = \pi w^4r(w)^2~{\rm d}s$. Her er ${\rm d}s = \dfrac{{\rm d}s}{{\rm d}w}{\rm d}w = \dfrac{1}{w_0}{\rm d}w$. Fra figuren er det lett å se at $r(w)^2+s(w)^2=1\Longrightarrow r(w) = \sqrt{1-w^2/w_0^2}$. Dette gir da $$Q = \left[\dots\right] = \dfrac{4\pi}{35}(a^2+b^2+c^2)^2$$
Bra jobba! Liker spesielt hvordan du utnyttet geometrien i problemet for å forenkle løsningen din. Eventuelt kan vi anvende et lurt triks og skrive integranden som et skalarprodukt: $$I = \iiint_D\left(ax + by + cz\right)^4\, \text{d}V = \iiint_D\left(\left(a,b,c\right)\cdot\left(x,y,z\right)\right)^4\, \text{d}V.$$ Dermed introduserer vi nye koordinater $\hat{x}, \hat{y}$ og $\hat{z}$ ved å rotere koordinatsystemet slik at $\hat{z}$-aksen er parallell med $\textbf{a} = (a,b,c)$. Vi introduserer enhetsvektorene $\mathbf{e}_{\hat{x}}, \mathbf{e}_{\hat{y}}$ og $\mathbf{e}_{\hat{z}}$ som positive retningsvektorer langs de nye koordinataksene. Merk at $\mathbf{a}\cdot\mathbf{e}_{\hat{x}} = \mathbf{a}\cdot\mathbf{e}_{\hat{y}} = 0.$ Ettersom integrasjonsområdet er sfærisk symmetrisk og Jacobianmatrisen har determinant $1$ får vi da at $$I = \iiint_D \left(\mathbf{a}\cdot\left(\hat{x}\mathbf{e}_{\hat{x}} + \hat{y}\mathbf{e}_{\hat{y}} + \hat{z}\mathbf{e}_{\hat{z}}\right)\right)^4\, \text{d} V = \iiint_D \left(\mathbf{a}\cdot\mathbf{e}_{\hat{z}}\hat{z}\right)^4 \, \text{d} V = \iiint_D \left(\mathbf{a}\cdot\frac{\hat{z}}{\lVert \mathbf{a} \rVert}\mathbf{a}\right)^4, \text{d} V$$
$$ = \lVert\mathbf{a}\rVert^4\iiint_D \hat{z}^4\, \text{d}V = \lVert\mathbf{a}\rVert^4 \int_{\varphi = 0}^{\pi}\int_{\theta = 0}^{2\pi}\int_{r=0}^1 r^6 \cos^4\varphi\sin\varphi\, \text{d}r\, \text{d}\theta\, \text{d}\varphi = \left[\dots\right] = \frac{4\pi}{35}\left(a^2 + b^2 + c^2\right)^2.$$
MatIsa skrev:Oppfølger: $\int \dfrac{{\rm d}x}{x(x+1)(x+2)\cdots (x+n)}$


Vi viser ved induksjon på $n$ at $$\frac{1}{x(x+1)\dots (x+n)} = \frac{1}{n!}\sum_{i=0}^n\beta_i^{(n)}\frac{1}{x+i},$$
der koeffisientene $\beta_i^{(n)}$ er bestemt av binomialuttrykket $$(u-v)^n = \sum_{i=0}^n\beta_i^{(n)} u^iv^{n-i}.$$ Det vil si, $\beta_i^{(n)} = (-1)^i{n \choose i}.$

$\mathbf{n=1}$: $$\frac{1}{x(x+1)} = \frac{1}{x} - \frac{1}{x+1}. \checkmark $$

Induksjon: Anta at formelen gjelder for $n$, der $n\geq 1$ er gitt. Da får vi at $$\frac{1}{x(x+1)\dots (x+n)(x+n+1)} = \frac{1}{x+n+1}\cdot\frac{1}{n!}\sum_{i=0}^n\beta_i^{(n)}\frac{1}{x+i} = \frac{1}{n!}\sum_{i=0}^n\beta_i^{(n)}\left(\frac{(n+1-i)^{-1}}{x+i} - \frac{(n+1-i)^{-1}}{x+n+1}\right).$$ Nå, $$\beta_i^{(n)}(n+1-i)^{-1} = (-1)^i{n \choose i}\frac{1}{n+1-i} = (-1)^i\frac{n!}{i!(n-i)!}\frac{1}{n+1-i} = (-1)^i\frac{n!}{i!(n+1-i)!}=\frac{1}{n+1}(-1)^i\frac{(n+1)!}{i!(n+1-i)!} = \frac{1}{n+1}\beta_i^{(n+1)},$$ og $$-\sum_{i=0}^n\beta_i^{(n)}\frac{1}{n+1-i} = -\sum_{i=0}^n(-1)^{i}\frac{n!}{i!(n-i)!}\frac{1}{n+1-i} = -\frac{1}{n+1}\sum_{i=0}^n{n+1 \choose i} = \frac{1}{n+1}\left[\sum_{i=0}^{n+1}{n+1 \choose i} - (-1)^{n+1}\right] = -\frac{1}{n+1}\left[(1-1)^{n+1} - (-1)^{n+1}\right] = \frac{1}{n+1}(-1)^{n+1} = \frac{1}{n+1}\beta_i^{(n+1)},$$ så $$\frac{1}{x(x+1)\dots(x+n)(x+n+1)} = \frac{1}{(n+1)!}\sum_{i=0}^{n+1}\beta_i^{(n+1)}\frac{1}{x+i},$$ som fullfører induksjonssteget. $\checkmark$

Dermed får vi at $$\int\frac{1}{x(x+1)\dots (x+n)}\, \text{d}x = \frac{1}{n!}\int\sum_{i=0}^n\beta_i^{(n)}\frac{1}{x+i}\, \text{d}x = \frac{1}{n!}\sum_{i=0}^n\beta_i^{(n)}\log | x+i | +C,$$ der $C\in\mathbb{R}$ er en integrasjonskonstant.

Oppfølger: Hvis $x\in\mathbb{R}$ er et reelt tall lar vi $\lfloor x \rfloor$ være det største heltallet mindre enn eller lik $x$. La $n\in\mathbb{N}$ være et naturlig tall. Finn $$\int_0^n \lfloor 2^x \rfloor \, \text{d}x.$$
Markus
Fermat
Fermat
Innlegg: 767
Registrert: 20/09-2016 13:48
Sted: NTNU

DennisChristensen skrev:Oppfølger: Hvis $x\in\mathbb{R}$ er et reelt tall lar vi $\lfloor x \rfloor$ være det største heltallet mindre enn eller lik $x$. La $n\in\mathbb{N}$ være et naturlig tall. Finn $$\int_0^n \lfloor 2^x \rfloor \, \text{d}x.$$
Plotter vi $f(x)=\lfloor 2^x \rfloor$ ser vi at $f(x)=1$ for $x \in [0,1)$, $f(x)=2$ for $x \in [\log_2 2, \log_2 3)$, $f(x)=3$ for $x \in [\log_2 3, \log_2 4)$. Mer generelt, for $n \in \mathbb{N}$ er $f(x)=n$ for $x \in [\log_2 n, \log_2 (n+1))$. Derfor blir $$\int_0^n \lfloor 2^x \rfloor \,\text{d}x = 1+ 2(\log_2 3 - \log_2 2) + 3(\log_2 4 - \log_2 3) + \dots + (2^{n}-1)(\log_2 (2^{n}) - \log_2 (2^{n} - 1)) \\ = 1 - 2\log_2 2 + 2\log_2 3 - 3\log_2 3 + 3\log_2 4 + \dots - (2^{n}-1)\log_2(2^n-1) +(2^{n}-1)\log_2 (2^{n}) \\ = -\log_2 2 - \log_2 3 - \log_2 4 - \dots - \log(2^{n}) + 2^{n}\log_2 (2^n) \\ = 2^{n}\log_2(2^n) - (\log_2 2 + \log_2 3 + \log_2 4 + \dots + \log_2(2n)) \\ = 2^n \cdot n - \log_2((2n)!)$$

En oppfølger i samme gate: $\int_0^n 2^{\lfloor x \rfloor} \, \text{d}x$, der $n \in \mathbb{N}$

Edit: da har jeg lagt til en oppfølger
MatIsa
Dirichlet
Dirichlet
Innlegg: 150
Registrert: 12/06-2013 12:09
Sted: Trondheim

Markus skrev:En oppfølger i samme gate: $\int_0^n 2^{\lfloor x \rfloor} \, \text{d}x$, der $n \in \mathbb{N}$
$\int_0^n 2^{\lfloor x\rfloor}~{\rm d}x = \sum_{i=0}^{n-1}\int_{i}^{i+1}2^{\lfloor x\rfloor}~{\rm d}x=\sum_{i=0}^{n-1}2^{i}\int_{i}^{i+1}~{\rm d}x=\sum_{i=0}^{n-1} 2^i = 2^n-1$

Oppfølger: $\int_0^\infty \dfrac{\sin^2(x)}{x^2}{\rm d}x$
Kay
Abel
Abel
Innlegg: 684
Registrert: 13/06-2016 19:23
Sted: Gløshaugen

MatIsa skrev:
Markus skrev:En oppfølger i samme gate: $\int_0^n 2^{\lfloor x \rfloor} \, \text{d}x$, der $n \in \mathbb{N}$
$\int_0^n 2^{\lfloor x\rfloor}~{\rm d}x = \sum_{i=0}^{n-1}\int_{i}^{i+1}2^{\lfloor x\rfloor}~{\rm d}x=\sum_{i=0}^{n-1}2^{i}\int_{i}^{i+1}~{\rm d}x=\sum_{i=0}^{n-1} 2^i = 2^n-1$

Oppfølger: $\int_0^\infty \dfrac{\sin^2(x)}{x^2}{\rm d}x$

Det er en trillion måter å regne den der på, bl.a. Fourier, kompleks og hva enn ikke.

Velger likevel å flekse litt laplace muskler fordi jeg har drevet med det en del i det siste, hehe...

Observer at [tex]\int_0^\infty \frac{sin^2(x)}{x^2}=\int_0^\infty \frac{sin(x)}{x}[/tex] (Kan bevise denne hvis du føler den trivialiserer problemet)

Vi kjenner at

[tex]\mathcal{L}\left \{ \frac{f(t)}{t} \right \}(s)=\int_s^\infty \mathcal{L}\left \{ f(t) \right \}(p)dp[/tex]

Ergo [tex]\int_0^\infty \mathcal{L}\left \{ \sin t \right \}(s)ds=\int_0^\infty \frac{1}{s^2+1}=\left [\arctan(s) \right ]_0^\infty=\frac{\pi}{2}[/tex]

Oppfølger [tex]\int_0^\infty xe^{-|x|}dx[/tex]
Markus
Fermat
Fermat
Innlegg: 767
Registrert: 20/09-2016 13:48
Sted: NTNU

Kay skrev:Oppfølger [tex]\int_0^\infty xe^{-|x|}dx[/tex]
På $\mathbb{R}^+$ er $xe^{-|x|} = xe^{-x}$ så $\int_0^\infty xe^{-|x|} \, \text{d}x = \int_0^\infty xe^{-x} \, \text{d}x = [-xe^{-x}]_0^\infty - \int_0^\infty e^{-x} = 1$ ved delvis integrasjon.
Oppfølger: $$\int_0^1 \frac{\arcsin \left (\frac{2x}{1+x^2} \right ) }{1+x^2} \, \text{d}x$$
Janhaa
Boltzmann
Boltzmann
Innlegg: 8552
Registrert: 21/08-2006 03:46
Sted: Grenland

Oppfølger: $$\int_0^1 \frac{\arcsin \left (\frac{2x}{1+x^2} \right ) }{1+x^2} \, \text{d}x$$
Cluet her vel å observere at:

$$\arcsin \left (\frac{2x}{1+x^2} \right ) =2u=2\arctan(x)$$
via:
[tex]x=\tan(u)[/tex]
etc...
slik at:
$$I=\int_0^1 \frac{\arcsin \left (\frac{2x}{1+x^2} \right ) }{1+x^2} \, \text{d}x$$


[tex]I=2\int_{0}^{1}\frac{\arctan(x)}{x^2+1}\,dx[/tex]
der
[tex]\theta =\arctan(x)\\ \\ d\theta = \frac{dx}{1+x^2}\\ \\ I=2\int_{0}^{\pi/4}\theta\,d\theta = \frac{\pi^2}{16}[/tex]
La verken mennesker eller hendelser ta livsmotet fra deg.
Marie Curie, kjemiker og fysiker.

[tex]\large\dot \rho = -\frac{i}{\hbar}[H,\rho][/tex]
Svar